What is System: Definition and 1000 Discussions

A system is a group of interacting or interrelated elements that act according to a set of rules to form a unified whole. A system, surrounded and influenced by its environment, is described by its boundaries, structure and purpose and expressed in its functioning. Systems are the subjects of study of systems theory.

View More On Wikipedia.org
  1. D

    Angular momentum of a system relative to a moving reference frame.

    I don't have too much of a clue of how to begin the problem. I first wrote the angular moementum of the system of particles: →M=∑mi(→ri×→vi)M→=∑mi(r→i×v→i). Then I know that the angular momentum from of the moving reference frame would have the velocity as the sum of the velocity of the frame...
  2. T

    Binary System in circular orbit: Separation distance between stars

    Distance is d=1/0.07 = 14.3 parsecs The Doppler shift of one star is, Δλ = 512 - 512.04 = -0.04 So the radical of the velocity of the star is = (-0.04/512) x (3.00 x 10^5 km/s) = 23.4km/s which is the same for both stars because they have the same mass. This is as far as I've got.
  3. W

    Root Locus of Negative Feedback System

    From my understanding, the root locus is only concerned with open loop gain. I figured this means you would ignore the negative feedback loop and calculate the root locus from just the plant's function Workings: zeros: -1 poles: 0, -2, -2, relative degree = 2 => 90-degree asymptotes meeting...
  4. L

    I What is the Basis of a Composite System?

    If I have a composite system, like a two particle system, for exemple, I can construct my Hilbert space as the tensor product of the hilbert spaces of these particles, and, if ##\{|A;m \rangle \}## and ##\{|B;n \rangle \}## are basis in these hilbert spaces, a basis in the total hilbert space is...
  5. B

    AQA physics paper On potential energy in mass spring system

    The spring constant of a helical spring is 28 N mñ1. A 0.40 kg mass is suspended from the spring and set into simple harmonic motion of amplitude 60 mm. i use the equation 1/2 change in extension times by force where the force i assume is mass times by 9.8 but the mark scheme uses 1/2 kx ^2 and...
  6. karush

    MHB -16.1 Find the general solution to the system of DE

    Find the general solution to the system of differential equations $\begin{cases} y'_1&=2y_1+y_2-y_3 \\ y'_2&=3y_2+y_3\\ y'_3&=3y_3 \end{cases}$ let $y(t)=\begin{bmatrix}{y_1(t)\\y_2(t)\\y_3(t)}\end{bmatrix} ,\quad A=\begin{bmatrix} 2 & 1 & -1 \\ 0 & 3 & 1 \\...
  7. jaewonjung

    Moment of Inertia of this combined system?

    Moment of inertia for a rod rotated around one end is 1/3ML^2 using the parallel axis theorem Moment of inertia for a sphere is 2/5 MR^2 Itotal=Isphere+Irod=2/5MR^2+1/3ML^2 However, the answer is 2/5MR^2+1/3ML^2+ML^2 Why is there an extra ML^2 added to the moment of inertia?
  8. karush

    MHB 29.1 give a system of fundamental solutions

    determine their general solution and give a system of fundamental solutions. use the different techniques of diagonal, diagonalizedable, or triangular. $\begin{cases} y'_1 & =3y_1 \\ y'_2 & =2y_2\end{cases}$ set matrix $A= \begin{pmatrix}0 &3\\0 &2\end{pmatrix}$ then find...
  9. karush

    MHB How to find the eigenvectors for a given matrix using the example in a textbook?

    Solve the system $$Y'=\begin{bmatrix}2 & 1 & 0 \\0 & 2 & 1 \\ 0 & 0 & 4 \end{bmatrix}Y$$ subtract $\lambda$ from the diagonal entries of the given matrix and take det: $$\left| \begin{array}{ccc} - \lambda + 2 & 1 & 0 \\ 0 & - \lambda + 2 & 1 \\ 0 & 0 & - \lambda + 4...
  10. R

    Does pressure in a compressed air system increase....

    does pressure in a compressed air system increase when going from a 2" air line to a 1" air line.
  11. karush

    MHB How to Solve System of Equations with Matrix Y

    nmh{383} 307.27.3 Solve the system $$Y'=\begin{bmatrix} 1 & 3 & -3 \\ 0 & 1 & 0 \\ 6 & 3 & -8 \end{bmatrix}Y$$ ok how do we get Y
  12. P

    Convert from a weighted system to a plate and bolt system

    Hi all, I need to convert a cooling fixture from a weighted system to a plate and bolt system. I'd like to use 4 - 10mm cap screws and a pressure plate to replace 7.82kg of weight. The weights are 190mm x 70mm of contact area. The pressure plate will have flanges to accommodate the bolts...
  13. Np14

    What is the angular acceleration of this cylindrical system?

    PART B ONLY: The cylinder undergoes torque when the mass m2 is removed: τNET = Iα = FNETr = 45α = FT(0.5) FT = 90α, therefore msystem = 90 kg After this step, I am not sure what to do. τ = ΣF = Fg - Ft = ma = (20 kg)(9.8 m/s2) - (90α) = 196 -...
  14. Np14

    What is the tension force for this system in rotational equilibrium?

    The system is in rotational equilibrium and therefore experiences no net torque, meaning all individual torques must add to zero. τNET = 0 = FFTsin(θ)L - FgL - Fg(L/2) τNET = 0 = FTsin30°(0.6?) - (0.5)(9.8)(0.6) - (2.0)(0.6/2) My only problem (I think) is figuring out what the length L is for...
  15. S

    Help with Bernoulli's equation for a central heating system

    so far I have found the velocity 1 and 2 by dividing the volume flow rate over the area which I got from pi x dia squared/4 my v1 = 1.01859m/s and v2= 2.82942m/s i have then figured out a pressure for the 15mm pipe which i got an answer of 2.71 bar however i am stuck on the rest of the question...
  16. K

    B Definition of coordinate system

    In light of the modern definition of what is a coordinate system, namely it's a pair (U, f) with U a region of a m-dimensional manifold, and f a bijection from U to ##\mathbb R^m##, can we say that the polar coordinates on ##\mathbb R^2## are a coordinate system? I was thinking about this and...
  17. Shivang kohlii

    Shm , calculation of amplitude of spring mass system

    Homework Statement In A spring mass system , the spring stretches 2 cm from its 's frelength when a force of 10 N is applied . This spring is stretched 10 cm from it's free length , when a body of mass m = 2 kg is attached to it and released from rest at time t = 0 . Find the A) force constant...
  18. Magellanic

    Thermo: Calculating the temperature drop from gas leaving a system

    Hi PF! I have a tricky problem that I'm trying to model, and none of the typical textbook examples cover this--or they only cover it tangentially--so I was hoping to get some insight here. (Couldn't find a "Thermodynamics" section of the forums so if there's a better place to post this, please...
  19. Leonardo Muzzi

    B Why can't we see dark matter in the solar system?

    If the distribution of elements in the universe is also the distribution of elements in the solar system for regular matter, and being dark matter so overwhelmingly prevalent in the universe, why can't we see it overwhelmingly in the solar system?
  20. P

    Fault in ungrounded system -> zero sequence voltage

    Hi. After some study, I came across symmetrical components. I found the attached schematic below on the internet, and will use it to explain my question. Question: Imagine that a single-line to Earth fault occur in one of the phases, i.e. resulting in an unbalanced system (we assume it was...
  21. P

    I Center of Mass of a jetpack-Earth System

    It is known that the center of mass of an isolated system will assume the same location no matter what internal forces there are (as long as no net external force occurs). My first question is if there exist two bodies of masses m1 and m2 in some space that has no friction, gravity, or...
  22. S

    Regression for a first order system

    Homework Statement I am carrying out a regression for diameter of a part Homework Equations Diameter = -0.0531052 + 0.0443237 * exp (-0.0103633 * 'Time elapsed') if diameter is -0.052 then can some one please calculate the value for time elapsed would you please explain the steps The...
  23. M

    A How can the stability of an ODE system be determined without solving it?

    Hi PF! Given the ODE system ##x'(t) = A(t) x(t)## where ##x## is a vector and ##A## a square matrix periodic, so that ##A(t) = A(T+t)##, would the following be a good way to solve the system's stability: fix ##t^*##. Then $$ \int \frac{1}{x} \, dx = \int A(t^*) \, dt \implies\\ x(t) =...
  24. F

    Maximum displacement in mass spring system

    Homework Statement A block is suspended by an ideal spring of the force constant K. If the block is pulled down by applying a constant force F and if maximum displacement of the block from its initial position of rest is X then, find the value of X. Homework Equations mg + F = XK + K(mg/K)...
  25. A

    Mass hanging from a 2-pulley system

    Homework Statement I have a question about the pulley problem in the attachment. Homework Equations This question can be answered using equilibrium of forces, namely Fx = 0 and Fy = 0. The Attempt at a Solution The answer key states that for the sum of the x-components, we should use the...
  26. KatrineRav

    Explain friction in a pulley system

    Hi! I'm working in a new area and I'm looking for inputs/suggestions for relevant factors to look into and possible ressource where I can read about this :) I did some tests in a pulley similar to the one below. I had a fixed weight at M2. I started with M1=M2 but added weights to M1 until...
  27. J

    Solving a system of two simultaneous trigonometric equations

    Homework Statement I need to solve a system of two equations for T and θ algebraic and with all the other parameters known. φ is equal to: Homework Equations The relevant equations are the two equations left of * in the image below The Attempt at a Solution I tried Gauss elimination but I...
  28. A

    MHB ็How to Solve the system of equation

    x_0\cos(\phi) = 2.78 x_0\sin(\phi)=2.78 \left( \frac{\gamma^2/2}{ \sqrt{10-\frac{\gamma^2}{4}}} \right) x_0e^{-15\gamma} \cos\left(30\sqrt{10-\frac{\gamma^2}{4}}-\phi\right)=1 I don't know awsner of \phi , x_,\gamma
  29. L

    What are the boundary conditions for this plate/ring system?

    Homework Statement [/B] I have a metal disc adhesively bonded at its edges to a piezoelectric ring. The piezoelectric ring vibrates radially which leads to the plate vibrating transversely. I am looking to work out the resonant frequency of the metal disc which I believe will depend on the...
  30. Mert Kerem Yavuz

    Motion of a system that has two masses and three pulleys

    Homework Statement Homework Equations F = ma The Attempt at a Solution I couldn't draw the freebody diagram. There's this weight of mass m, which is mg, downwards from m and there's tension T to upwards. This T affects mass M in the +x direction, but how could i find out the normal force...
  31. cyboman

    Boeing How Safe is the Boeing 737 Max's MCAS System?

    Hi, I have a question regarding the tragic crash of the latest 737 Max. Is it not a huge error in the flight laws and the MCAS software to execute a nose down maneuver at any altitude? Should the system not have a rule to prohibit such a maneuver below a minimum altitude threshold? I'm also...
  32. M

    Work exchanged by a system, and internal forces' work

    Homework Statement Hi to everyone, I'm studying thermodynamics, especially I'm focused on the work exchanged by a system. Book's explanation says that when the system received a force from the environment [F(e-s)] (on its surface) it reacts with a force from the system to the environment...
  33. M

    Difference between system subchannel and CFD codes

    Hi everyone, I am researching how numerical simulations have evolved over the years in nuclear reactors for assessing the thermal hydraulics inside the reactor core. I have found vague information in regards to the three different main numerical techniques but want to learn more. So far...
  34. X

    I Take a limit in this 2 equation system of 1st order ODEs

    Hello, I'm having a problem with this system. Ignore the physics. I have the feeling it should be tremendously easy... but I can't figure it out. I don't know how to extract it from the pdf so I'll post just the these 2 pages. https://ufile.io/39ovq The equations are (1.14) and (1.15), the...
  35. D

    Photosensor-Brake System for a Small Plastic Car

    Hello, I am attempting to expand my physical understanding of some engineering principles I’ve learned during my first semester of ChemE. I am making a small plastic car that will be propelled with a weak fan. I am attempting something similar to a ChemE project car. Is there a way I can use a...
  36. iVenky

    Power of noise after passing through a system h(t)

    **Reposting this again, as I was asked to post this on a homework forum** 1. Homework Statement Hi, I am trying to solve this math equation (that I found on a paper) on finding the variance of a noise after passing through an LTI system whose impulse response is h(t) X(t) is the input noise...
  37. D

    Pulley system, find the acceleration and tension

    Homework Statement Given the picture: Make the free body diagrams of each body and the pulley. Remember that there are two different strings therefore there are two different tensions.Assuming that the mobile of mass M starts from rest and has a displacement D at a time t, and the bodywith...
  38. confused student

    System of Differential Equations

    Homework Statement (It should be noted that the actual problem has specific values associated with a, b, and c. However, at this point I'm trying to find a method to solve the problem rather than a specific solution). Homework Equations The Attempt at a Solution When I was trying to solve...
  39. iVenky

    I White noise & 1/f noise after a system h(t)

    Hi, I am trying to solve this math equation on finding the variance of a noise after passing through a system whose impulse response is h(t) X is the input noise of the system and Y is the output noise after system h(t) if let's say variance of noise Y is σy2=∫∫Rxx(u,v)h(u)h(v)dudv where...
  40. M

    Twin spring system with a mass in between (X and Y direction)

    Hello my fellow scientists! This is my first post on this forum, but most certainly not my last. I am relatively new to physics. I mean, i understand the basics. I would like to make it my hobby. You know, really find a passion for the physics. BTW, i am from the Netherlands, so please do not...
  41. Zack K

    Potential difference in a 2 disk system (Capacitor)

    Homework Statement A capacitor consists of two large metal disks placed a distance ##s## apart. The radius of each disk is R ## (R \gg s)## and the thickness of each disk is ##t##. The disk on the left has a net charge of ##+Q## and the disk on the right has a net charge of ##-Q##. Calculate...
  42. K

    Find Velocity of Particle for System of Charges

    Homework Statement Two charges, ##-q_1## and ##q_2## are fixed in the vacuum and separated by a distance ##a##. What should be the velocity ##v## of a particle with mass ##m## and charge ##q##, traveling from an infinitely far point along the line which unites ##q_1## and ##q_2## in order to...
  43. W

    Work Check: Heat transfer between reservoir and small system

    Homework Statement Could someone look through my work? The parts where I wrote (??) are steps I am especially unsure about. Many thanks in advance. A large reservoir at temperature ##T_r## is placed in thermal contact with a small system at temperature ##T##. They end up at temperature...
  44. H

    Question about a system in thermodynamic equilibrium

    What are the precise conditions for thermodynamic equilibrium? I know that a system in thermodynamic equilibrium must have constant temperature and that there can be no net macroscopic flow of energy or matter. However, is it possible for there to be a system in equilibrium that has a spatially...
  45. J

    PV system with an array of 1500 watts

    Hello all, I am in deep water here based on my very fundamental understanding of physics generally and solar design specifically. I am 83 years, homeless, and have access to land with one building, wood heat, non-potable site water, and nothing else. I want to make a PV system with an array...
  46. T

    I The Universe as an isolated system

    Can the universe be treated as a thermodynamic isolated system? And why?
  47. EEristavi

    Solving a System of Equations via the Matrix Method

    I have equation system: x + y + z - a*k = 0 -b*x + y + z = 0 -c*y + z = 0 -d*x + y = 0 where: a, b, c, d = const. Have to find: x, y, z, k Attempt of solution: I create Matrix A with coefficients; Matrix B - Solutions (Zeros) and Matrix X - variables. When I try to use Cramer's rule -...
  48. cookiemnstr510510

    I Determining if the system is consistent

    Hello all, I have the "correct" answer from Chegg. However, I am not satisfied that I really understand. Heres the problem: Determine if the system is consistent. Do not completely solve the system. 2X1-4X4=-10 3X2+3X3=0 X3+4X4=-1 -3X1+2X2+3X3+X4=5 Here is my attempt: I first write the system...
  49. Zack K

    Final potential difference of a 2 capacitor system

    Homework Statement An isolated parallel-plate capacitor of area ##A_1## with an air gap of length ##s_1## is charged up to a potential difference ##\Delta V_1## A second parallel-plate capacitor, initially uncharged, has an area ##A_2## and a gap of length ##s_2## filled with plastic whose...
Back
Top